Sie sind auf Seite 1von 35

[G.R. No. 128195.

October 3, 2001]

ELIZABETH LEE and PACITA YU LEE, HON. JUDGE JOSE D. ALOVERA,* Presiding Judge, Regional
Trial Court, Branch 17, Roxas City, THE REGISTER OF DEEDS OF ROXAS
CITY, petitioners, vs. REPUBLIC OF THE PHILIPPINES, represented by THE DIRECTOR OF
LANDS AND THE ADMINISTRATOR, LAND REGISTRATION AUTHORITY and THE HON. COURT
OF APPEALS,* respondents.

DECISION
PARDO, J.:

The case under consideration is a petition for review on certiorari of the decision[1] of the Court of Appeals
nullifying that of the Regional Trial Court, Roxas City, in Reconstitution Case No. R-1928,[2] pertaining to Lot
398, CapizCadastre, covered by Original Certificate of Title No. 3389.
Sometime in March 1936, Rafael, Carmen, Francisco, Jr., Ramon, Lourdes, Mercedes, Concepcion,
Mariano, Jose, Loreto, Manuel, Rizal and Jimmy, all surnamed Dinglasan sold to Lee Liong, a Chinese citizen,
a parcel of land with an approximate area of 1,631 square meters, designated as Lot 398 and covered by
Original Certificate of Title No. 3389, situated at the corner of Roxas Avenue and Pavia Street, Roxas City.[3]
However, in 1948, the former owners filed with the Court of First Instance, Capiz an action against the
heirs of Lee Liong for annulment of sale and recovery of land.[4] The plaintiffs assailed the validity of the sale
because of the constitutional prohibition against aliens acquiring ownership of private agricultural land,
including residential, commercial or industrial land. Rebuffed in the trial court and the Court of Appeals,
plaintiffs appealed to the Supreme Court. On June 27, 1956, the Supreme Court ruled thus:

“… granting the sale to be null and void and can not give title to the vendee, it does not necessarily follow
therefrom that the title remained in the vendor, who had also violated the constitutional prohibition, or that he
(vendor) has the right to recover the title of which he has divested himself by his act in ignoring the
prohibition. In such contingency another principle of law sets in to bar the equally guilty vendor from
recovering the title which he had voluntarily conveyed for a consideration, that of pari delicto.”[5]

On July 1, 1968, the same former owners Rafael A. Dinglasan, together with Francisco, Carmen, Ramon,
Lourdes, Mercedes, Concepcion, Mariano, Jose, Loreto, Rizal, Jimmy, and Jesse Dinglasan filed with the
Court of First Instance, Capiz an action for recovery of the same parcel of land. [6] Citing the case of Philippine
Banking Corporation v. Lui She,[7] they submitted that the sale to Lee Liong was null and void for being
violative of the Constitution. On September 23, 1968, the heirs of Lee Liong filed with the trial court a motion to
dismiss the case on the ground of res judicata.[8] On October 10, 1968, and November 9, 1968, the trial court
denied the motion.[9] The heirs of Lee Liong elevated the case to the Supreme Court by petition for
certiorari. On April 22, 1977, the Supreme Court annulled the orders of the trial court and directed it to dismiss
the case, holding that the suit was barred by res judicata.[10]
On September 7, 1993, Elizabeth Manuel-Lee and Pacita Yu Lee filed with the Regional Trial Court,
Roxas City a petition for reconstitution of title of Lot No. 398 of the CapizCadastre, formerly covered by Original
Certificate of Title No. 3389 of the Register of Deeds of Roxas City. [11] Petitioners alleged that they were the
widows of the deceased Lee Bing Hoo and Lee Bun Ting, who were the heirs of Lee Liong, the owner of the
lot. Lee Liong died intestate in February 1944. On June 30, 1947, Lee Liong’s widow, Ang Chia, and his two
sons, Lee Bun Ting and Lee Bing Ho, executed an extra-judicial settlement of the estate of Lee Liong,
adjudicating to themselves the subject parcel of land.[12] Petitioner Elizabeth Lee acquired her share in Lot No.
398 through an extra-judicial settlement and donation executed in her favor by her deceased husband Lee
Bing Hoo. Petitioner Pacita Yu Lee acquired her share in the same lot by succession from her deceased
husband Lee Bun Ting, as evidenced by a deed of extra-judicial settlement.[13]
Previously, on December 9, 1948, the Register of Deeds, Capiz, Salvador Villaluz, issued a certification
that a transfer certificate of title over the property was issued in the name of Lee Liong. [14] However, the
records of the Register of Deeds, Roxas City were burned during the war. Thus, as heretofore stated, on
September 7, 1968, petitioners filed a petition for reconstitution of title.
On June 10, 1994, the Regional Trial Court, Roxas City, Branch 17, ordered the reconstitution of the lost
or destroyed certificate of title in the name of Lee Liong on the basis of an approved plan and technical
description.[15] The dispositive portion of the trial court’s decision reads thus:

“WHEREFORE, in reiteration, the Register of Deeds for the City of Roxas is ordered to reconstitute the lost or
destroyed certificate of title in the name of Lee Liong, deceased, of Roxas City, with all the conditions stated in
paragraph 2 of this decision. This decision shall become final after the lapse of thirty (30) days from receipt by
the Register of Deeds and by the Commissioner of LRA of a notice of such judgment without any appeal
having been filed by any of such officials.

“SO ORDERED.

“Given at Roxas City, Philippines,

“June 10, 1994.

“JOSE O. ALOVERA
“Judge”[16]

On August 18, 1994, the Clerk of Court, Regional Trial Court, Roxas City, Branch 17 issued an Entry of
Judgment.[17]
On January 25, 1995, the Solicitor General filed with the Court of Appeals a petition for annulment of
judgment in Reconstitution Case No. 1928, alleging that the Regional Trial Court,Roxas City had no jurisdiction
over the case.[18] The Solicitor General contended that the petitioners were not the proper parties in the
reconstitution of title, since their predecessor-in-interest Lee Liong did not acquire title to the lot because he
was a Chinese citizen and was constitutionally not qualified to own the subject land.
On April 30, 1996, the Court of Appeals promulgated its decision declaring the judgment of reconstitution
void.[19]
On May 24, 1996, Elizabeth Manuel-Lee and Pacita Yu Lee filed with the Court of Appeals a motion for
reconsideration of the decision.[20] On February 18, 1997, the Court of Appeals denied the motion.[21]
Hence, this petition.[22]
Petitioners submitted that the Solicitor General was estopped from seeking annulment of the judgment of
reconstitution after failing to object during the reconstitution proceedings before the trial court, despite due
notice. Petitioners alleged that the Solicitor General merely acted on the request of private and politically
powerful individuals who wished to capitalize on the prime location of the subject land.
Petitioners emphasized that the ownership of the land had been settled in two previous cases of the
Supreme Court, where the Court ruled in favor of their predecessor-in-interest, Lee Liong. Petitioners also
pointed out that they acquired ownership of the land through actual possession of the lot and their consistent
payment of taxes over the land for more than sixty years.
On the other hand, the Solicitor General submitted that the decision in the reconstitution case was void;
otherwise, it would amount to circumventing the constitutional proscription against aliens acquiring ownership
of private or public agricultural lands.
We grant the petition.
The reconstitution of a certificate of title denotes restoration in the original form and condition of a lost or
destroyed instrument attesting the title of a person to a piece of land. [23] The purpose of the reconstitution of
title is to have, after observing the procedures prescribed by law, the title reproduced in exactly the same way it
has been when the loss or destruction occurred.[24]
In this case, petitioners sought a reconstitution of title in the name of Lee Liong, alleging that the transfer
certificate of title issued to him was lost or destroyed during World War II. All the documents recorded and
issued by the Register of Deeds, Capiz, which include the transfer certificate of title issued in the name of Lee
Liong, were all destroyed during the war. The fact that the original of the transfer certificate of title was not in
the files of the Office of the Register of Deeds did not imply that a transfer certificate of title had not been
issued.[25] In the trial court proceedings, petitioners presented evidence proving the sale of the land from the
Dinglasans to Lee Liong and the latter’s subsequent possession of the property in the concept of owner. Thus,
the trial court, after examining all the evidence before it, ordered the reconstitution of title in the name of Lee
Liong.
However, there is a question as to whether Lee Liong has the qualification to own land in the Philippines.
The sale of the land in question was consummated sometime in March 1936, during the effectivity of the
1935 Constitution. Under the 1935 Constitution,[26] aliens could not acquire private agricultural lands, save in
cases of hereditary succession.[27] Thus, Lee Liong, a Chinese citizen, was disqualified to acquire the land in
question.[28]
The fact that the Court did not annul the sale of the land to an alien did not validate the transaction, for it
was still contrary to the constitutional proscription against aliens acquiring lands of the public or private
domain. However, the proper party to assail the illegality of the transaction was not the parties to the
transaction.[29] “In sales of real estate to aliens incapable of holding title thereto by virtue of the provisions of
the Constitution both the vendor and the vendee are deemed to have committed the constitutional violation and
being thus in pari delicto the courts will not afford protection to either party.”[30] The proper party to assail the
sale is the Solicitor General. This was what was done in this case when the Solicitor General initiated an
action for annulment of judgment of reconstitution of title. While it took the Republic more than sixty years to
assert itself, it is not barred from initiating such action. Prescription never lies against the State.[31]
Although ownership of the land cannot revert to the original sellers, because of the doctrine of pari delicto,
the Solicitor General may initiate an action for reversion or escheat of the land to the State, subject to other
defenses, as hereafter set forth.[32]
In this case, subsequent circumstances militate against escheat proceedings because the land is now in
the hands of Filipinos. The original vendee, Lee Liong, has since died and the land has been inherited by his
heirs and subsequently their heirs, petitioners herein. Petitioners are Filipino citizens, a fact the Solicitor
General does not dispute.
The constitutional proscription on alien ownership of lands of the public or private domain was intended to
protect lands from falling in the hands of non-Filipinos. In this case, however, there would be no more public
policy violated since the land is in the hands of Filipinos qualified to acquire and own such land. “If land is
invalidly transferred to an alien who subsequently becomes a citizen or transfers it to a citizen, the flaw in the
original transaction is considered cured and the title of the transferee is rendered valid.” [33] Thus, the
subsequent transfer of the property to qualified Filipinos may no longer be impugned on the basis of the
invalidity of the initial transfer.[34] The objective of the constitutional provision to keep our lands in Filipino hands
has been achieved.
Incidentally, it must be mentioned that reconstitution of the original certificate of title must be based on an
owner’s duplicate, secondary evidence thereof, or other valid sources of the title to be reconstituted. [35] In this
case, reconstitution was based on the plan and technical description approved by the Land Registration
Authority.[36] This renders the order of reconstitution void for lack of factual support. [37] A judgment with
absolutely nothing to support it is void.[38]
As earlier mentioned, a reconstitution of title is the re-issuance of a new certificate of title lost or destroyed
in its original form and condition.[39] It does not pass upon the ownership of the land covered by the lost or
destroyed title.[40] Any change in the ownership of the property must be the subject of a separate suit. [41] Thus,
although petitioners are in possession of the land, a separate proceeding is necessary to thresh out the issue
of ownership of the land.
WHEREFORE, the Court REVERSES and SETS ASIDE the decision of the Court of Appeals in CA-G. R.
SP No. 36274. In lieu thereof, the Court sets aside the order of reconstitution of title in Reconstitution Case
No. R-1928, Regional Trial Court, Roxas City, and dismisses the petition, without prejudice.
No costs.
SO ORDERED.

[G.R. No. 153883. January 13, 2004]

REPUBLIC OF THE PHILIPPINES, petitioner, vs. CHULE Y. LIM, respondent.

DECISION
YNARES-SANTIAGO, J.:

This petition for review on certiorari under Rule 45 of the Rules of Court stemmed from a petition for
correction of entries under Rule 108 of the Rules of Court filed by respondent Chule Y. Lim with the Regional
Trial Court of Lanao del Norte, Branch 4, docketed as Sp. Proc. No. 4933.
In her petition, respondent claimed that she was born on October 29, 1954 in Buru-an, Iligan City. Her
birth was registered in Kauswagan, Lanaodel Norte but the Municipal Civil Registrar of Kauswagan transferred
her record of birth to Iligan City. She alleged that both her Kauswagan and Iligan City records of birth have
four erroneous entries, and prays that they be corrected.
The trial court then issued an Order,[1] which reads:

WHEREFORE, finding the petition to be sufficient in form and substance, let the hearing of this case be set on
December 27, 1999 before this Court, Hall of Justice, Rosario Heights, Tubod, Iligan City at 8:30 o’clock in the
afternoon at which date, place and time any interested person may appear and show cause why the petition
should not be granted.

Let this order be published in a newspaper of general circulation in the City of Iligan and the Province of Lanao
del Norte once a week for three (3) consecutive weeks at the expense of the petitioner.

Furnish copies of this order the Office of the Solicitor General at 134 Amorsolo St., LegaspiVill., Makati City
and the Office of the Local Civil Registrar of Iligan City at Quezon Ave., Pala-o, Iligan City.

SO ORDERED.

During the hearing, respondent testified thus:


First, she claims that her surname “Yu” was misspelled as “Yo”. She has been using “Yu” in all her school
records and in her marriage certificate.[2] She presented a clearance from the National Bureau of Investigation
(NBI)[3] to further show the consistency in her use of the surname “Yu”.
Second, she claims that her father’s name in her birth record was written as “Yo Diu To (Co Tian)” when it
should have been “Yu Dio To (Co Tian).”
Third, her nationality was entered as Chinese when it should have been Filipino considering that her father
and mother never got married. Only her deceased father was Chinese, while her mother is Filipina. She
claims that her being a registered voter attests to the fact that she is a Filipino citizen.
Finally, it was erroneously indicated in her birth certificate that she was a legitimate child when she should
have been described as illegitimate considering that her parents were never married.
Placida Anto, respondent’s mother, testified that she is a Filipino citizen as her parents were both Filipinos
from Camiguin. She added that she and her daughter’s father were never married because the latter had a
prior subsisting marriage contracted in China.
In this connection, respondent presented a certification attested by officials of the local civil registries of
Iligan City and Kauswagan, Lanaodel Norte that there is no record of marriage between Placida Anto and Yu
Dio To from 1948 to the present.
The Republic, through the City Prosecutor of Iligan City, did not present any evidence although it actively
participated in the proceedings by attending hearings and cross-examining respondent and her witnesses.
On February 22, 2000, the trial court granted respondent’s petition and rendered judgment as follows:

WHEREFORE, the foregoing premises considered, to set the records of the petitioner straight and in their
proper perspective, the petition is granted and the Civil Registrar of Iligan City is directed to make the following
corrections in the birth records of the petitioner, to wit:

1. Her family name from “YO” to “YU”;

2. Her father’s name from “YO DIU TO (CO TIAN)” to “YU DIOTO (CO TIAN)”;

3. Her status from “legitimate” to “illegitimate” by changing “YES” to “NO” in answer to the
question “LEGITIMATE?”; and,

4. Her citizenship from “Chinese” to “Filipino”.

SO ORDERED.[4]

The Republic of the Philippines appealed the decision to the Court of Appeals which affirmed the trial
court’s decision.[5]
Hence, this petition on the following assigned errors:
I

THE COURT OF APPEALS ERRED IN ORDERING THE CORRECTION OF THE CITIZENSHIP OF


RESPONDENT CHULE Y. LIM FROM “CHINESE” TO “FILIPINO” DESPITE THE FACT THAT RESPONDENT
NEVER DEMONSTRATED ANY COMPLIANCE WITH THE LEGAL REQUIREMENTS FOR ELECTION OF
CITIZENSHIP.

II

THE COURT OF APPEALS ERRED IN ALLOWING RESPONDENT TO CONTINUE USING HER FATHER’S
SURNAME DESPITE ITS FINDING THAT RESPONDENT IS AN ILLEGITIMATE CHILD.[6]

To digress, it is just as well that the Republic did not cite as error respondent’s recourse to Rule 108 of the
Rules of Court to effect what indisputably are substantial corrections and changes in entries in the civil
register. To clarify, Rule 108 of the Revised Rules of Court provides the procedure for cancellation or
correction of entries in the civil registry. The proceedings under said rule may either be summary or adversary
in nature. If the correction sought to be made in the civil register is clerical, then the procedure to be adopted
is summary. If the rectification affects the civil status, citizenship or nationality of a party, it is deemed
substantial, and the procedure to be adopted is adversary. This is our ruling in Republic v. Valencia[7] where
we held that even substantial errors in a civil registry may be corrected and the true facts established under
Rule 108 provided the parties aggrieved by the error avail themselves of the appropriate adversary
proceeding. An appropriate adversary suit or proceeding is one where the trial court has conducted
proceedings where all relevant facts have been fully and properly developed, where opposing counsel have
been given opportunity to demolish the opposite party’s case, and where the evidence has been thoroughly
weighed and considered.[8]
As likewise observed by the Court of Appeals, we take it that the Republic’s failure to cite this error
amounts to a recognition that this case properly falls under Rule 108 of the Revised Rules of Court considering
that the proceeding can be appropriately classified as adversarial.
Instead, in its first assignment of error, the Republic avers that respondent did not comply with the
constitutional requirement of electing Filipino citizenship when she reached the age of majority. It cites Article
IV, Section 1(3) of the 1935 Constitution, which provides that the citizenship of a legitimate child born of a
Filipino mother and an alien father followed the citizenship of the father, unless, upon reaching the age of
majority, the child elected Philippine citizenship.[9] Likewise, the Republic invokes the provision in Section 1 of
Commonwealth Act No. 625, that legitimate children born of Filipino mothers may elect Philippine citizenship
by expressing such intention “in a statement to be signed and sworn to by the party concerned before any
officer authorized to administer oaths, and shall be filed with the nearest civil registry. The said party shall
accompany the aforesaid statement with the oath of allegiance to the Constitution and the Government of the
Philippines.”[10]
Plainly, the above constitutional and statutory requirements of electing Filipino citizenship apply only
to legitimate children. These do not apply in the case of respondent who was concededly an illegitimate child,
considering that her Chinese father and Filipino mother were never married. As such, she was not required to
comply with said constitutional and statutory requirements to become a Filipino citizen. By being an illegitimate
child of a Filipino mother, respondent automatically became a Filipino upon birth. Stated differently, she is a
Filipino since birth without having to elect Filipino citizenship when she reached the age of majority.
In Ching, Re: Application for Admission to the Bar,[11] citing In re Florencio Mallare,[12] we held:

Esteban Mallare, natural child of Ana Mallare, a Filipina, is therefore himself a Filipino, and no other act would
be necessary to confer on him all the rights and privileges attached to Philippine citizenship (U.S. vs.
OngTianse, 29 Phil. 332; Santos Co vs. Government of the Philippine Islands, 42 Phil. 543; Serra vs. Republic,
L-4223, May 12, 1952; SyQuimsuan vs. Republic, L-4693, Feb. 16, 1953; Pitallano vs. Republic, L-5111, June
28, 1954). Neither could any act be taken on the erroneous belief that he is a non-Filipino divest him of the
citizenship privileges to which he is rightfully entitled.[13]

This notwithstanding, the records show that respondent elected Filipino citizenship when she reached the
age of majority. She registered as a voter in Misamis Oriental when she was 18 years old.[14] The exercise of
the right of suffrage and the participation in election exercises constitute a positive act of election of Philippine
citizenship.[15]
In its second assignment of error, the Republic assails the Court of Appeals’ decision in allowing
respondent to use her father’s surname despite its finding that she is illegitimate.
The Republic’s submission is misleading. The Court of Appeals did not allow respondent to use her
father’s surname. What it did allow was the correction of her father’s misspelled surname which she has been
using ever since she can remember. In this regard, respondent does not need a court pronouncement for her
to use her father’s surname.
We agree with the Court of Appeals when it held:

Firstly, Petitioner-appellee is now 47 years old. To bar her at this time from using her father’s surname which
she has used for four decades without any known objection from anybody, would only sow
confusion. Concededly, one of the reasons allowed for changing one’s name or surname is to avoid confusion.

Secondly, under Sec. 1 of Commonwealth Act No. 142, the law regulating the use of aliases, a person is
allowed to use a name “by which he has been known since childhood.”

Thirdly, the Supreme Court has already addressed the same issue. In Pabellar v. Rep. of the Phils.,[16] we
held:
Section 1 of Commonwealth Act No. 142, which regulates the use of aliases, allows a person to use a name
“by which he has been known since childhood” (Lim Hok Albano v. Republic, 104 Phil. 795; People v.
UyJuiPio, 102 Phil. 679; Republic v. Tañada, infra). Even legitimate children cannot enjoin the illegitimate
children of their father from using his surname (De Valencia v. Rodriguez, 84 Phil. 222). [17]

While judicial authority is required for a change of name or surname, [18] there is no such requirement for
the continued use of a surname which a person has already been using since childhood.[19]
The doctrine that disallows such change of name as would give the false impression of family relationship
remains valid but only to the extent that the proposed change of name would in great probability cause
prejudice or future mischief to the family whose surname it is that is involved or to the community in
general.[20] In this case, the Republic has not shown that the Yu family in China would probably be prejudiced
or be the object of future mischief. In respondent’s case, the change in the surname that she has been using
for 40 years would even avoid confusion to her community in general.
WHEREFORE, in view of the foregoing, the instant petition for review is DENIED. The decision of the
Court of Appeals in CA-G.R. CV No. 68893 dated May 29, 2002, is AFFIRMED. Accordingly, the Civil Registrar
of Iligan City is DIRECTED to make the following corrections in the birth record of respondent Chule Y. Lim, to
wit:

1. Her family name from “YO” to “YU”;

2. Her father’s name from “YO DIU TO (CO TIAN)” to “YU DIOTO (CO TIAN)”;

3. Her status from “legitimate” to “illegitimate” by changing “YES” to “NO” in answer to the question
“LEGITIMATE?”; and,

4. Her citizenship from “Chinese” to “Filipino”.

SO ORDERED.
Republic v. Lim (January 13, 2004) Digest

Facts:
-Chule Y. Lim filed a petition for correction of entries under Rule 108 of the Rules of Court with the Regional
Trial Court of Lanao del Norte.
-She claimed that she was born on 29 October 1954 in Buru-an, Iligan City. Her birth was registered in
Kauswagan, Lanaodel Norte but the Municipal Civil Registrar of Kauswagan transferred her record of birth to
Iligan City.
-the Court finding the petition sufficient in form and substance ordered the publication of the hearing of the
petition.
-during the hearing, Lim testifies that:
-her surname "Yu" was misspelled as "Yo". She has been using "Yu" in all her school records and in
her marriage certificate.2 She presented a clearance from the National Bureau of Investigation (NBI) to further
show the consistency in her use of the surname "Yu".
-she claims that her father’s name in her birth record was written as "Yo Diu To (Co Tian)" when it
should have been "Yu Dio To (Co Tian).
-her nationality was entered as Chinese when it should have been Filipino considering that her father
and mother never got married. Only her deceased father was Chinese, while her mother is Filipina. She claims
that her being a registered voter attests to the fact that she is a Filipino citizen.
-it was erroneously indicated in her birth certificate that she was a legitimate child when she should
have been described as illegitimate considering that her parents were never married. She also presented a
certification attested by officials of the local civil registries of Iligan City and Kauswagan, Lanaodel Norte that
there is no record of marriage between Placida Anto and Yu Dio To from 1948 to the present.
-the Court then granted the petition but the Republic appealed to the CA.
-the Republic claims that Lim never complied with the legal requirement in electing her citizenship
-the Republic assails the Court of Appeals’ decision in allowing respondent to use her father’s surname despite
its finding that she is illegitimate
-the questioned corrections were:
1. Her family name from "YO" to "YU";
2. Her father’s name from "YO DIU TO (CO TIAN)" to "YU DIOTO (CO TIAN)";
3. Her status from "legitimate" to "illegitimate" by changing "YES" to "NO" in answer to the question
"LEGITIMATE?"; and,
4. Her citizenship from "Chinese" to "Filipino".
Issue:
WON CA erred in granting Lim’s petition to correct her civil registry records, especially citizenship.
Held:
Petition to review DENIED
Ratio:
The Republic avers that respondent did not comply with the constitutional requirement of electing Filipino
citizenship when she reached the age of majority. It cites Article IV, Section 1(3) of the 1935 Constitution,
which provides that the citizenship of a legitimate child born of a Filipino mother and an alien father followed
the citizenship of the father, unless, upon reaching the age of majority, the child elected Philippine citizenship.
Plainly, the above constitutional and statutory requirements of electing Filipino citizenship apply only
to legitimate children. These do not apply in the case of respondent who was concededly an illegitimate child,
considering that her Chinese father and Filipino mother were never married. As such, she was not required to
comply with said constitutional and statutory requirements to become a Filipino citizen. By being an illegitimate
child of a Filipino mother, respondent automatically became a Filipino upon birth. Stated differently, she is a
Filipino since birth without having to elect Filipino citizenship when she reached the age of majority.
The Republic’s submission is misleading. The Court of Appeals did not allow respondent to use her father’s
surname. What it did allow was the correction of her father’s misspelled surname which she has been using
ever since she can remember. In this regard, respondent does not need a court pronouncement for her to use
her father’s surname.
While judicial authority is required for a change of name or surname, there is no such requirement for the
continued use of a surname which a person has already been using since childhood.
The doctrine that disallows such change of name as would give the false impression of family relationship
remains valid but only to the extent that the proposed change of name would in great probability cause
prejudice or future mischief to the family whose surname it is that is involved or to the community in general. In
this case, the Republic has not shown that the Yu family in China would probably be prejudiced or be the
object of future mischief. In respondent’s case, the change in the surname that she has been using for 40
years would even avoid confusion to her community in general.

TECSON VS. COMELEC


G.R. No. 161434, March 3 2004

FACTS:
Respondent Ronald Allan Kelly Poe, also known as Fernando Poe, Jr. (FPJ) filed his certificate of candidacy
on 31 December 2003 for the position of President of the Republic of the Philippines in the forthcoming
national elections. In his certificate of candidacy, FPJ, representing himself to be a natural-born citizen of
the Philippines, stated his name to be "Fernando Jr.," or "Ronald Allan" Poe, his date of birth to be 20 August
1939 and his place of birth to be Manila.

Petitioner Fornier filed before the COMELEC a petition to disqualify FPJ and cancel his certificate of candidacy
by claiming that FPJ is not a natural-born Filipino citizen, his parents were foreigners: his mother, Bessie
Kelley Poe, was an American, and his father, Allan Poe, was a Spanish national, being the son of Lorenzo
Pou, a Spanish subject.
The COMELEC dismissed the petition for lack of merit.

ISSUE:
Whether or not FPJ is a natural-born citizen of the Philippines.

HELD:
Section 2, Article VII, of the 1987 Constitution expresses:
No person may be elected President unless he is a natural-born citizen of the Philippines, a registered voter,
able to read and write, at least forty years of age on the day of the election, and a resident of the Philippines for
at least ten years immediately preceding such election.
Natural-born citizens are those who are citizens of the Philippines from birth without having to perform any act
to acquire or perfect their Philippine citizenship. Based on the evidence presented which the Supreme consider
as viable is the fact that the death certificate of Lorenzo Poe, father of Allan Poe, who in turn was the father of
private respondent Fernando Poe, Jr. indicates that he died on September 11, 1954 at the age of 84 years, in
San Carlos, Pangasinan. Evidently, in such death certificate, the residence of Lorenzo Poe was stated to be
San Carlos, Pangansinan. In the absence of any evidence to the contrary, it should be sound to conclude, or at
least to presume, that the place of residence of a person at the time of his death was also his residence before
death. Considering that the allegations of petitioners are not substantiated with proof and since Lorenzo Poe
may have been benefited from the “en masse Filipinization” that the Philippine Bill had effected in 1902, there
is no doubt that Allan Poe father of private respondent Fernando Poe, Jr. was a Filipino citizen. And, since the
latter was born on August 20, 1939, governed under 1935 Constitution, which constitution considers as citizens
of the Philippines those whose fathers are citizens of the Philippines, Fernando Poe, Jr. was in fact a natural-
born citizen of the Philippines regardless of whether or not he is legitimate or illegitimate.

G.R. No. 161434 March 3, 2004


MARIA JEANETTE C. TECSON and FELIX B. DESIDERIO, JR. vs.COMELEC, FPJ and VICTORINO X.
FORNIER,

G.R. No. 161634 March 3, 2004


ZOILO ANTONIO VELEZ vs.FPJ

G. R. No. 161824 March 3, 2004


VICTORINO X. FORNIER, vs. HON. COMMISSION ON ELECTIONS and FPJ

Facts:
Petitioners sought for respondent Poe’s disqualification in the presidential elections for having allegedly
misrepresented material facts in his (Poe’s) certificate of candidacy by claiming that he is a natural Filipino
citizen despite his parents both being foreigners. Comelec dismissed the petition, holding that Poe was a
Filipino Citizen. Petitioners assail the jurisdiction of the Comelec, contending that only the Supreme Court may
resolve the basic issue on the case under Article VII, Section 4, paragraph 7, of the 1987 Constitution.
Issue:
Whether or not it is the Supreme Court which had jurisdiction.
Whether or not Comelec committed grave abuse of discretion in holding that Poe was a Filipino citizen.
Ruling:
1.) The Supreme Court had no jurisdiction on questions regarding “qualification of a candidate” for the presidency
or vice-presidency before the elections are held.
"Rules of the Presidential Electoral Tribunal" in connection with Section 4, paragraph 7, of the 1987
Constitution, refers to “contests” relating to the election, returns and qualifications of the "President" or "Vice-
President", of the Philippines which the Supreme Court may take cognizance, and not of "candidates" for
President or Vice-President before the elections.
2.) Comelec committed no grave abuse of discretion in holding Poe as a Filipino Citizen.

The 1935 Constitution on Citizenship, the prevailing fundamental law on respondent’s birth, provided
that among the citizens of the Philippines are "those whose fathers are citizens of the Philippines."

Tracing respondent’s paternal lineage, his grandfather Lorenzo, as evidenced by the latter’s death certificate
was identified as a Filipino Citizen. His citizenship was also drawn from the presumption that having died in
1954 at the age of 84, Lorenzo would have been born in 1980. In the absence of any other evidence,
Lorenzo’s place of residence upon his death in 1954 was presumed to be the place of residence prior his
death, such that Lorenzo Pou would have benefited from the "en masse Filipinization" that the Philippine Bill
had effected in 1902. Being so, Lorenzo’s citizenship would have extended to his son, Allan---respondent’s
father.

Respondent, having been acknowledged as Allan’s son to Bessie, though an American citizen, was a Filipino
citizen by virtue of paternal filiation as evidenced by the respondent’s birth certificate. The 1935 Constitution on
citizenship did not make a distinction on the legitimacy or illegitimacy of the child, thus, the allegation of
bigamous marriage and the allegation that respondent was born only before the assailed marriage had no
bearing on respondent’s citizenship in view of the established paternal filiation evidenced by the public
documents presented.

But while the totality of the evidence may not establish conclusively that respondent FPJ is a natural-born
citizen of the Philippines, the evidence on hand still would preponderate in his favor enough to hold that he
cannot be held guilty of having made a material misrepresentation in his certificate of candidacy in violation of
Section 78, in relation to Section 74 of the Omnibus Election Code.

BAR MATTER No. 914 October 1, 1999

RE: APPLICATION FOR ADMISSION TO THE PHILIPPINE BAR,

vs.

VICENTE D. CHING, applicant.

RESOLUTION

KAPUNAN, J.:

Can a legitimate child born under the 1935 Constitution of a Filipino mother and an alien father validly elect
Philippine citizenship fourteen (14) years after he has reached the age of majority? This is the question sought
to be resolved in the present case involving the application for admission to the Philippine Bar of Vicente D.
Ching.

The facts of this case are as follows:

Vicente D. Ching, the legitimate son of the spouses Tat Ching, a Chinese citizen, and Prescila A. Dulay, a
Filipino, was born in Francia West, Tubao, La Union on 11 April 1964. Since his birth, Ching has resided in the
Philippines.

On 17 July 1998, Ching, after having completed a Bachelor of Laws course at the St. Louis University in
Baguio City, filed an application to take the 1998 Bar Examinations. In a Resolution of this Court, dated 1
September 1998, he was allowed to take the Bar Examinations, subject to the condition that he must submit to
the Court proof of his Philippine citizenship.

In compliance with the above resolution, Ching submitted on 18 November 1998, the following documents:

1. Certification, dated 9 June 1986, issued by the Board of Accountancy of the Professional
Regulations Commission showing that Ching is a certified public accountant;

2. Voter Certification, dated 14 June 1997, issued by Elizabeth B. Cerezo, Election Officer of the
Commission on Elections (COMELEC) in Tubao La Union showing that Ching is a registered
voter of the said place; and

3. Certification, dated 12 October 1998, also issued by Elizabeth B. Cerezo, showing that Ching
was elected as a member of the Sangguniang Bayan of Tubao, La Union during the 12 May
1992 synchronized elections.

On 5 April 1999, the results of the 1998 Bar Examinations were released and Ching was one of the successful
Bar examinees. The oath-taking of the successful Bar examinees was scheduled on 5 May 1999. However,
because of the questionable status of Ching's citizenship, he was not allowed to take his oath. Pursuant to the
resolution of this Court, dated 20 April 1999, he was required to submit further proof of his citizenship. In the
same resolution, the Office of the Solicitor General (OSG) was required to file a comment on Ching's petition
for admission to the bar and on the documents evidencing his Philippine citizenship.

The OSG filed its comment on 8 July 1999, stating that Ching, being the "legitimate child of a Chinese father
and a Filipino mother born under the 1935 Constitution was a Chinese citizen and continued to be so, unless
upon reaching the age of majority he elected Philippine citizenship" 1 in strict compliance with the provisions of
Commonwealth Act No. 625 entitled "An Act Providing for the Manner in which the Option to Elect Philippine
Citizenship shall be Declared by a Person Whose Mother is a Filipino Citizen." The OSG adds that "(w)hat he
acquired at best was only an inchoate Philippine citizenship which he could perfect by election upon reaching
the age of majority." 2 In this regard, the OSG clarifies that "two (2) conditions must concur in order that the
election of Philippine citizenship may be effective, namely: (a) the mother of the person making the election
must be a citizen of the Philippines; and (b) said election must be made upon reaching the age of
majority." 3 The OSG then explains the meaning of the phrase "upon reaching the age of majority:"

The clause "upon reaching the age of majority" has been construed to mean a reasonable time
after reaching the age of majority which had been interpreted by the Secretary of Justice to be
three (3) years (VELAYO, supra at p. 51 citing Op., Sec. of Justice No. 70, s. 1940, Feb. 27,
1940). Said period may be extended under certain circumstances, as when a (sic) person
concerned has always considered himself a Filipino (ibid., citing Op. Nos. 355 and 422, s. 1955;
3, 12, 46, 86 and 97, s. 1953). But in Cuenco, it was held that an election done after over seven
(7) years was not made within a reasonable time.

In conclusion, the OSG points out that Ching has not formally elected Philippine citizenship and, if ever he
does, it would already be beyond the "reasonable time" allowed by present jurisprudence. However, due to the
peculiar circumstances surrounding Ching's case, the OSG recommends the relaxation of the standing rule on
the construction of the phrase "reasonable period" and the allowance of Ching to elect Philippine citizenship in
accordance with C.A. No. 625 prior to taking his oath as a member of the Philippine Bar.

On 27 July 1999, Ching filed a Manifestation, attaching therewith his Affidavit of Election of Philippine
Citizenship and his Oath of Allegiance, both dated 15 July 1999. In his Manifestation, Ching states:

1. I have always considered myself as a Filipino;

2. I was registered as a Filipino and consistently declared myself as one in my school records
and other official documents;

3. I am practicing a profession (Certified Public Accountant) reserved for Filipino citizens;

4. I participated in electoral process[es] since the time I was eligible to vote;

5. I had served the people of Tubao, La Union as a member of the Sangguniang Bayan from
1992 to 1995;

6. I elected Philippine citizenship on July 15, 1999 in accordance with Commonwealth Act No.
625;

7. My election was expressed in a statement signed and sworn to by me before a notary public;

8. I accompanied my election of Philippine citizenship with the oath of allegiance to the


Constitution and the Government of the Philippines;

9. I filed my election of Philippine citizenship and my oath of allegiance to (sic) the Civil
Registrar of Tubao La Union, and

10. I paid the amount of TEN PESOS (Ps. 10.00) as filing fees.

Since Ching has already elected Philippine citizenship on 15 July 1999, the question raised is whether he has
elected Philippine citizenship within a "reasonable time." In the affirmative, whether his citizenship by election
retroacted to the time he took the bar examination.

When Ching was born in 1964, the governing charter was the 1935 Constitution. Under Article IV, Section 1(3)
of the 1935 Constitution, the citizenship of a legitimate child born of a Filipino mother and an alien father
followed the citizenship of the father, unless, upon reaching the age of majority, the child elected Philippine
citizenship. 4 This right to elect Philippine citizenship was recognized in the 1973 Constitution when it provided
that "(t)hose who elect Philippine citizenship pursuant to the provisions of the Constitution of nineteen hundred
and thirty-five" are citizens of the Philippines. 5 Likewise, this recognition by the 1973 Constitution was carried
over to the 1987 Constitution which states that "(t)hose born before January 17, 1973 of Filipino mothers, who
elect Philippine citizenship upon reaching the age of majority" are Philippine citizens. 6 It should be noted,
however, that the 1973 and 1987 Constitutional provisions on the election of Philippine citizenship should not
be understood as having a curative effect on any irregularity in the acquisition of citizenship for those covered
by the 1935 Constitution. 7 If the citizenship of a person was subject to challenge under the old charter, it
remains subject to challenge under the new charter even if the judicial challenge had not been commenced
before the effectivity of the new Constitution. 8

C.A. No. 625 which was enacted pursuant to Section 1(3), Article IV of the 1935 Constitution, prescribes the
procedure that should be followed in order to make a valid election of Philippine citizenship. Under Section 1
thereof, legitimate children born of Filipino mothers may elect Philippine citizenship by expressing such
intention "in a statement to be signed and sworn to by the party concerned before any officer authorized to
administer oaths, and shall be filed with the nearest civil registry. The said party shall accompany the aforesaid
statement with the oath of allegiance to the Constitution and the Government of the Philippines."

However, the 1935 Constitution and C.A. No. 625 did not prescribe a time period within which the election of
Philippine citizenship should be made. The 1935 Charter only provides that the election should be made "upon
reaching the age of majority." The age of majority then commenced upon reaching twenty-one (21) years. 9 In
the opinions of the Secretary of Justice on cases involving the validity of election of Philippine citizenship, this
dilemma was resolved by basing the time period on the decisions of this Court prior to the effectivity of the
1935 Constitution. In these decisions, the proper period for electing Philippine citizenship was, in turn, based
on the pronouncements of the Department of State of the United States Government to the effect that the
election should be made within a "reasonable time" after attaining the age of majority. 10 The phrase
"reasonable time" has been interpreted to mean that the election should be made within three (3) years from
reaching the age of
majority. 11 However, we held in Cuenco vs. Secretary of Justice, 12 that the three (3) year period is not an
inflexible rule. We said:

It is true that this clause has been construed to mean a reasonable period after reaching the
age of majority, and that the Secretary of Justice has ruled that three (3) years is the reasonable
time to elect Philippine citizenship under the constitutional provision adverted to above, which
period may be extended under certain circumstances, as when the person concerned has
always considered himself a Filipino. 13

However, we cautioned in Cuenco that the extension of the option to elect Philippine citizenship is not
indefinite:

Regardless of the foregoing, petitioner was born on February 16, 1923. He became of age on
February 16, 1944. His election of citizenship was made on May 15, 1951, when he was over
twenty-eight (28) years of age, or over seven (7) years after he had reached the age of majority.
It is clear that said election has not been made "upon reaching the age of majority." 14

In the present case, Ching, having been born on 11 April 1964, was already thirty-five (35) years old when he
complied with the requirements of C.A. No. 625 on 15 June 1999, or over fourteen (14) years after he had
reached the age of majority. Based on the interpretation of the phrase "upon reaching the age of majority,"
Ching's election was clearly beyond, by any reasonable yardstick, the allowable period within which to exercise
the privilege. It should be stated, in this connection, that the special circumstances invoked by Ching, i.e., his
continuous and uninterrupted stay in the Philippines and his being a certified public accountant, a registered
voter and a former elected public official, cannot vest in him Philippine citizenship as the law specifically lays
down the requirements for acquisition of Philippine citizenship by election.

Definitely, the so-called special circumstances cannot constitute what Ching erroneously labels as informal
election of citizenship. Ching cannot find a refuge in the case of In re: Florencio Mallare, 15 the pertinent portion
of which reads:

And even assuming arguendo that Ana Mallare were (sic) legally married to an alien, Esteban's
exercise of the right of suffrage when he came of age, constitutes a positive act of election of
Philippine citizenship. It has been established that Esteban Mallare was a registered voter as of
April 14, 1928, and that as early as 1925 (when he was about 22 years old), Esteban was
already participating in the elections and campaigning for certain candidate[s]. These acts are
sufficient to show his preference for Philippine citizenship.16

Ching's reliance on Mallare is misplaced. The facts and circumstances obtaining therein are very different from
those in the present case, thus, negating its applicability. First, EstebanMallare was born before the effectivity
of the 1935 Constitution and the enactment of C.A. No. 625. Hence, the requirements and procedures
prescribed under the 1935 Constitution and C.A. No. 625 for electing Philippine citizenship would not be
applicable to him. Second, the ruling in Mallare was an obiter since, as correctly pointed out by the OSG, it was
not necessary for Esteban Mallare to elect Philippine citizenship because he was already a Filipino, he being a
natural child of a Filipino mother. In this regard, the Court stated:

Esteban Mallare, natural child of Ana Mallare, a Filipina, is therefore himself a Filipino, and no
other act would be necessary to confer on him all the rights and privileges attached to Philippine
citizenship (U.S. vs. OngTianse, 29 Phil. 332; Santos Co vs. Government of the Philippine
Islands, 42 Phil. 543, Serra vs. Republic, L-4223, May 12, 1952, SyQuimsuan vs. Republic, L-
4693, Feb. 16, 1953; Pitallano vs. Republic, L-5111, June 28, 1954). Neither could any act be
taken on the erroneous belief that he is a non-filipino divest him of the citizenship privileges to
which he is rightfully entitled. 17

The ruling in Mallare was reiterated and further elaborated in Co vs. Electoral Tribunal of the House of
Representatives, 18 where we held:

We have jurisprudence that defines "election" as both a formal and an informal process.

In the case of In re: Florencio Mallare (59 SCRA 45 [1974]), the Court held that the exercise of
the right of suffrage and the participation in election exercises constitute a positive act of
election of Philippine citizenship. In the exact pronouncement of the Court, we held:

Esteban's exercise of the right of suffrage when he came of age constitutes a


positive act of Philippine citizenship. (p. 52: emphasis supplied)

The private respondent did more than merely exercise his right of suffrage. He has established his life here in
the Philippines.

For those in the peculiar situation of the respondent who cannot be excepted to have elected
Philippine citizenship as they were already citizens, we apply the In Re Mallare rule.

xxxxxxxxx

The filing of sworn statement or formal declaration is a requirement for those who still have to
elect citizenship. For those already Filipinos when the time to elect came up, there are acts of
deliberate choice which cannot be less binding. Entering a profession open only to Filipinos,
serving in public office where citizenship is a qualification, voting during election time, running
for public office, and other categorical acts of similar nature are themselves formal
manifestations for these persons.

An election of Philippine citizenship presupposes that the person electing is an alien. Or his
status is doubtful because he is a national of two countries. There is no doubt in this case about
Mr. Ong's being a Filipino when he turned twenty-one (21).

We repeat that any election of Philippine citizenship on the part of the private respondent would
not only have been superfluous but it would also have resulted in an absurdity. How can a
Filipino citizen elect Philippine citizenship? 19

The Court, like the OSG, is sympathetic with the plight of Ching. However, even if we consider the special
circumstances in the life of Ching like his having lived in the Philippines all his life and his consistent belief that
he is a Filipino, controlling statutes and jurisprudence constrain us to disagree with the recommendation of the
OSG. Consequently, we hold that Ching failed to validly elect Philippine citizenship. The span of fourteen (14)
years that lapsed from the time he reached the age of majority until he finally expressed his intention to elect
Philippine citizenship is clearly way beyond the contemplation of the requirement of electing "upon reaching
the age of majority." Moreover, Ching has offered no reason why he delayed his election of Philippine
citizenship. The prescribed procedure in electing Philippine citizenship is certainly not a tedious and
painstaking process. All that is required of the elector is to execute an affidavit of election of Philippine
citizenship and, thereafter, file the same with the nearest civil registry. Ching's unreasonable and unexplained
delay in making his election cannot be simply glossed over.

Philippine citizenship can never be treated like a commodity that can be claimed when needed and suppressed
when convenient. 20 One who is privileged to elect Philippine citizenship has only an inchoate right to such
citizenship. As such, he should avail of the right with fervor, enthusiasm and promptitude. Sadly, in this case,
Ching slept on his opportunity to elect Philippine citizenship and, as a result. this golden privilege slipped away
from his grasp.

IN VIEW OF THE FOREGOING, the Court Resolves to DENY Vicente D. Ching's application for admission to
the Philippine Bar.

SO ORDERED.

IN RE: VICENTE CHING


Bar Matter No. 914, October 1, 1999

FACTS:
Vicente D. Ching, the legitimate son of the spouses Tat Ching, a Chinese citizen, and Prescila A. Dulay, a
Filipino, was born in Francia West, Tubao, La Union on 11 April 1964. Since his birth, Ching has resided in the
Philippines.

After having completed a Bachelor of Laws course at the St. Louis University in Baguio City, Ching filed an
application to take the 1998 Bar Examinations. In a Resolution of this Court, he was allowed to take the
Bar Examinations, subject to the condition that he must submit to the Court proof of his Philippine citizenship.
On November 1998, he submitted the important documents in compliance with the said resolution.

During the 1998 Bar Examinations, Ching was one of the successfulexaminees but he was not allowed to take
the oath because of his questionable citizenship status. Pursuant to the resolution of this Court, he was
required to submit further proof of his citizenship. In the same resolution, the Office of the Solicitor General
(OSG) was required to file a comment on Ching's petition for admission to the bar and on the documents
evidencing his Philippine citizenship.

In their comment, the OSG points out that Ching has not formally elected Philippine citizenship and, if ever he
does, it would already be beyond the "reasonable time" allowed by present jurisprudence.

Ching then filed a Manifestation, attaching therewith his Affidavit of Election of Philippine Citizenship and his
Oath of Allegiance, both dated 15 July 1999.

ISSUE:
Whether or not he has elected Philippine citizenship within a "reasonable time."

HELD:
The Court holds that Ching failed to validly elect Philippinecitizenship. The span of fourteen (14) years that
lapsed from the time he reached the age of majority until he finally expressed his intention to elect
Philippine citizenship is clearly way beyond the contemplation of the requirement of electing "upon reaching
the age of majority." Moreover, Ching has offered no reason why he delayed his election of
Philippine citizenship. The prescribed procedure in electing Philippine citizenship is certainly not a tedious and
painstaking process. All that is required of the elector is to execute an affidavit of election of
Philippine citizenship and, thereafter, file the same with the nearest civil registry. Ching's unreasonable and
unexplained delay in making his election cannot be simply glossed over.

Philippine citizenship can never be treated like a commodity that can be claimed when needed and suppressed
when convenient. One who is privileged to elect Philippine citizenship has only an inchoate right to
such citizenship. As such, he should avail of the right with fervor, enthusiasm and promptitude. Sadly, in this
case, Ching slept on his opportunity to elect Philippine citizenship and, as a result this golden privilege slipped
away from his grasp. Therefore, the Court Resolves to DENY Vicente D. Ching's application for admission to
the Philippine Bar.

CO VS. HRET
G.R. No. 92191-92, July 30, 1991

FACTS:
On May 11, 1987, the congressional election for the second district of Northern Samar was held. Among
the candidates who vied for the position of representative in the second legislative district ofNorthern Samar
are the petitioners, SixtoBalinquit and Antonio Co and the private respondent, Jose Ong, Jr. Respondent Ong
was proclaimed the duly elected representative of the second district ofNorthern Samar.

The petitioners filed election protests against the private respondent alleging that Jose Ong, Jr. is not a natural
born citizen of the Philippines and not a resident of the second district of NorthernSamar.

The House of Representatives Electoral Tribunal (HRET) declared respondent Ong is a natural born Filipino
citizen and a resident of Laoang, Northern Samar for voting purposes.

ISSUES:
1. Whether or not respondent is a natural born Filipino and a resident of Laoang, Northern Samar.
2. Whether or not the HRET committed grave abuse of authority in the exercise of its powers.
HELD:
1. The Court affirmed the decision of HRET that respondent is a natural born Filipino and a resident of
Laoang, Northern Samar. The respondent traces his natural born citizenship through his mother, not through
the citizenship of his father. The citizenship of the father is relevant only to determine whether or not the
respondent "chose" to be a Filipino when he came of age. At that time and up to the present, both mother and
father were Filipinos. Respondent Ong could not have elected any other citizenship unless he first formally
renounced Philippine citizenship in favor of a foreign nationality. Unlike other persons faced with a problem
of election, there was no foreign nationality of his father which he could possibly have chosen.

2. The Court declared that HRET did not commit any grave abuse of discretion. The same issue of natural-
born citizenship has already been decided by the Constitutional Convention of 1971 and by the
BatasangPambansa convened by authority of the Constitutiondrafted by that Convention. Emil Ong, full blood
brother of the respondent, was declared and accepted as a natural born citizen by both bodies.

G.R. No. L-1663 March 31, 1948

FLORETINA VILLAHERMOSA, petitioner-appellant,


vs.
THE COMMISSIONER OF IMMIGRATION, respondent-appellee.

Victoriano V. Valle for appellant.


First Assistant Solicitor General Roberto A. Gianzon and Solicitor Francisco Carreon for appellee.

BENGZON, J.:

This is an appeal from the order of Honorable SoteroRodas, Judge of the Manila Court of First Instance,
denying the writ of habeas corpus requested by Florentina Villahermosa on behalf of her son Delfin Co, who is
under detention by the immigration authorities for purposes of deportation.
In the night of March 24, 1947, a party of sixty-nine Chinese landed clandestinely on the shores of Sto.
Domingo, Ilocos Sur, in an attempt to evade our immigration laws. Leading them was Delfin Co, a young man,
18 years old, born in Paniqui, Tarlac, of a Chinese father named Co Sut, alias Yu Kui, and Florentina
Villahermosa his wife. Co Suy died in July 1940, and inn February 1946, Delfin left the Philippines for China on
board S/S Cushman as a Chinese repatriate, in company with his relative Co Chi Pe. However, due to financial
difficulties in China, he took steps to return; but having met a Chines (Co Soon Tiong), who informed him of a
plan to smuggle their compatriots into this country, he agreed to lead the party to Ilocos Sur where his mother
had relatives who could render valuable assistance. The voyage was undertaken; but unfortunately, the
immigrants were discovered and apprehended immediately after arrival, and on the 27th day of March, Delfin
Co was examined by the Commissioner of Immigration. Formal investigation of the case began on April 10,
1947. Four days later, the corresponding board recommended that said Delfin Co be deported to China as a
Chinese citizen. The Commissioner of Immigration agreed with the board, and acting on this recommendation,
rendered a decision ordering the deportation of Delfin Co.

It appears that on April 29, 1947, Florentina Villahermosa, after knowing the apprehension of her son Delfin,
filed in the civil registry of Tarlac under Commonwealth Act No. 63 an oath of allegiance for the purpose of
resuming her Philippine citizenship which she had lost upon her marriage to Co Suy. On the strength of such
reacquisition of Philippine citizenship by Florentina, it was contended before the immigration authorities that
Delfin, being a minor, followed the citizenship of his mother, and was a national not subject to deportation.
These contentions were overruled. They were repeated before the court of first instance in this habeas
corpus proceeding and were likewise rejected. Appellant stresses he same defense.

There are two reasons why Delfin Co must be returned to China. First, he is not now a Filipino citizen; and
second, granting that he is, at the time he entered this country from China he was a Chinese subject to
deportation, and any subsequent change in his status can not erase the taint of his unlawful, surreptitious
entry.

Section 1 of Article IV of the Constitution enumerates those who are citizens of the Philippines, as follows:

(1) Those who are citizens of the Philippine Islands at the time of the adoption of the Constitution.

(2) Those born in the Philippine Islands of foreign parents who, before adoption of this Constitution, had
been elected to public office in the Philippine Islands.

(3) Those whose fathers are citizens of the Philippines.

(4) Those whose mothers are citizens of the Philippines and upon reaching the age of majority, elect
Philippine citizenship.

(5) Those who are naturalized in accordance with law.

DelfinCo's claim to citizenship can only be predicated, if at all, on paragraph 4 of the above section. But, being
a minor he has not had the opportunity to elect Philippine citizenship, and therefore he is as yet an alien, his
father being a Chinese.

We have heretofore held1 that, after the Constitution, mere birth in the Philippines of a Chinese father and
Filipino mother does not ipso facto confer Philippine citizenship and that jus sanguinis instead off jus soli is the
predominating factor on questions of citizenship, thereby rendering obsolete the decision in Roa vs. Collector
of Customs, 23 Phil., and U.S. vs. Lim Bin, 36 Phil., and similar cases on which petitioner's counsel relies.

Nevertheless, it is contended that Florentina Villahermosa being a Filipina, Delfin Co, should likewise be a
Filipino. Commonwealth Act No. 63 does not provide that upon repatriation of a Filipina her children acquire
Philippine citizenship. It would be illogical to consider Delfin as repatriated like his mother, because he never
was a Filipino citizen and could not have reacquired such citizenship.
While his Chinese father lived, Delfin was not a Filipino. His mother was not a Filipina; she was Chinese. After
the death of such father, Villahermosa continued to be a Chinese, until she reacquired her Filipino citizenship
in April, 1947. After that reacquisition Delfin could claim that his mother was a Filipina within the meaning of
paragraph 4, section 1 of Article IV of the Constitution; but, according to that same Organic Act, he had to elect
Philippine citizenship upon attaining his majority. Until he becomes of age and makes the election, he is the
Chinese citizen that he was at the time of his father's demise2.

It does not help petitioner's case to assert that as a mother she has a right to retain custody of her minor son
and to keep him here. Where such son has violated the immigration laws and rendered himself liable to
deportation no rule or principle should frustrate the Government's action by the interposition of the mother's
right to custody. This consideration becomes stronger where, as in this case, the re-assumption of Philippine
citizenship by Villahermosa has all the earmarks of an attempt to impede the banishment of Delfin Co, who by
the way, besides being guilty of violating our laws, has not shown any signs of eagerness to adopt our ways of
life.

This petition is moreover to be denied on the strength of precedents heretofore established, because Delfin
was a Chinese when he arrived here; and any posterior change of status can not affect the legality of his
detention for purposes of deportation.

In Juan Co vs. Rafferty, 14 Phil., 235, a Chinaman claimed the right to enter the Islands, and being refused by
the customs officials, gave bond that he would present himself for deportation if the claim were disallowed.
While under bond, he was adopted as a son by another Chinaman domiciled herein, in legal form. Held: he is
subject to deportation, because such adoption had no effect upon his right to enter or remain in the Islands.
This Court said that the status of an immigrant and his right to stay here is to be determined as of the time of
his entry (U.S. vs. Ju-Toy, 198 U.S., 253, 263) and that he could not do afterwards anything to render valid
what was originally an illegal entry.

A Chinese person, not a merchant at the time he applies to enter the Islands, will not be permitted to
remain here upon the theory that he became a merchant during the time he was waiting for the decision
of the proper authorities, (Tan Guam Sien vs. Collector of Customs, 31 Phil., 56.) (See also, U.S. vs.
Chan Sam, 17 Phil., 448)

We declare that Delfin Co is not now a Filipino. We also declare that he having entered this country
surreptitiously is subject to deportation.

The decision of the lower court denying his petition for habeas corpus is affirmed. With costs.

BENGSON vs. HRET and CRUZ


G.R. No. 142840
May 7, 2001
FACTS: The citizenship of respondent Cruz is at issue in this case, in view of the constitutional requirement
that “no person shall be a Member of the House of Representatives unless he is a natural-born citizen.”
Cruz was a natural-born citizen of the Philippines. He was born in Tarlac in 1960 of Filipino parents. In 1985,
however, Cruz enlisted in the US Marine Corps and without the consent of the Republic of the Philippines, took
an oath of allegiance to the USA. As a Consequence, he lost his Filipino citizenship for under CA No. 63 [(An
Act Providing for the Ways in Which Philippine Citizenship May Be Lost or Reacquired (1936)] section 1(4), a
Filipino citizen may lose his citizenship by, among other, “rendering service to or accepting commission in the
armed forces of a foreign country.”

Whatever doubt that remained regarding his loss of Philippine citizenship was erased by his naturalization as a
U.S. citizen in 1990, in connection with his service in the U.S. Marine Corps.
In 1994, Cruz reacquired his Philippine citizenship through repatriation under RA 2630 [(An Act Providing for
Reacquisition of Philippine Citizenship by Persons Who Lost Such Citizenship by Rendering Service To, or
Accepting Commission In, the Armed Forces of the United States (1960)]. He ran for and was elected as the
Representative of the 2nd District of Pangasinan in the 1998 elections. He won over petitioner Bengson who
was then running for reelection.

Subsequently, petitioner filed a case for Quo Warranto Ad Cautelam with respondent HRET claiming that Cruz
was not qualified to become a member of the HOR since he is not a natural-born citizen as required under
Article VI, section 6 of the Constitution.
HRET rendered its decision dismissing the petition for quo warranto and declaring Cruz the duly elected
Representative in the said election.
ISSUE: WON Cruz, a natural-born Filipino who became an American citizen, can still be considered a natural-
born Filipino upon his reacquisition of Philippine citizenship.
HELD: petition dismissed
YES
Filipino citizens who have lost their citizenship may however reacquire the same in the manner provided by
law. C.A. No. 63 enumerates the 3 modes by which Philippine citizenship may be reacquired by a former
citizen:
1. by naturalization,
2. by repatriation, and
3. by direct act of Congress.
**
Repatriation may be had under various statutes by those who lost their citizenship due to:
1. desertion of the armed forces;
2. services in the armed forces of the allied forces in World War II;
3. service in the Armed Forces of the United States at any other time,
4. marriage of a Filipino woman to an alien; and
5. political economic necessity
Repatriation results in the recovery of the original nationality This means that a naturalized Filipino who lost his
citizenship will be restored to his prior status as a naturalized Filipino citizen. On the other hand, if he was
originally a natural-born citizen before he lost his Philippine citizenship, he will be restored to his former status
as a natural-born Filipino.

R.A. No. 2630 provides:


Sec 1. Any person who had lost his Philippine citizenship by rendering service to, or accepting commission in,
the Armed Forces of the United States, or after separation from the Armed Forces of the United States,
acquired United States citizenship, may reacquire Philippine citizenship by taking an oath of allegiance to the
Republic of the Philippines and registering the same with Local Civil Registry in the place where he resides or
last resided in the Philippines. The said oath of allegiance shall contain a renunciation of any other citizenship.
Having thus taken the required oath of allegiance to the Republic and having registered the same in the Civil
Registry of Magantarem, Pangasinan in accordance with the aforecited provision, Cruz is deemed to have
recovered his original status as a natural-born citizen, a status which he acquired at birth as the son of a
Filipino father. It bears stressing that the act of repatriation allows him to recover, or return to, his original
status before he lost his Philippine citizenship.

REPUBLIC VS. DE LA ROSA


G.R. No. 104654, June 6 1994, 232 SCRA 785

FACTS:
This is a petition for certiorari under Rule 45 of the Revised Rules ofCourt in relation to R.A. No. 5440 and
Section 25 of the Interim Rules, filed by the Republic of the Philippines: (1) to annul the Decision
of the Regional Trial Court, Branch 28, Manila, which re-admitted private respondent as a Filipino citizen under
the Revised Naturalization Law (C.A. No. 63 as amended by C.A. No. 473); and (2) to nullify the oath
of allegiance taken by private respondent on February 27, 1992.
Petitioner then filed a petition on September 20, 1991 for naturalization captioned to be re-admitted as citizen
of the Philippines.

On January 14, 1992, private respondent filed a "Motion to SetHearing Ahead of Schedule, that it shall be
done on January instead of having it on March, " where he manifested his intention to run for public office in
the May 1992 elections. The motion was granted and the hearing was moved on February.

On February 27, respondent Judge rendered the assailed Decision and held that Petitioner JUAN G.
FRIVALDO, is re-admitted as a citizen of the Republic of the Philippines by naturalization, thereby vesting
upon him, all the rights and privileges of a natural born Filipino citizen.

The Solicitor General subsequently interposed a timely appeal directly with the Supreme Court.

ISSUE:
Whether or not the petitioner was duly re-admitted to his citizenshipas Filipino.

HELD:
No. The supreme court ruled that Private respondent is declared NOT a citizen of the Philippines and therefore
disqualified from continuing to serve as governor of the Province of Sorsogon. He is ordered to vacate his
office and to surrender the same to the Vice-Governor of the Province of Sorsogon once this decision becomes
final and executory. The proceedings of the trial court was marred by the following irregularities: (1)
the hearing of the petition was set ahead of the scheduled date of hearing, without a publication of the order
advancing the date of hearing, and the petition itself; (2) thepetition was heard within six months from the last
publication of the petition; (3) petitioner was allowed to take his oath of allegiancebefore the finality of the
judgment; and (4) petitioner took his oath of allegiance without observing the two-year waiting period.

G.R. No. L-35947 October 20, 1992

REPUBLIC OF THE PHILIPPINES, movant-appellee,


vs.
WILLIAM LI YAO, petitioner-appellant.

ROMERO, J.:

This is an appeal from the order of the then Court of First Instance of Manila over twenty years ago, or on July
22, 1971, cancelling the certificate of naturalization of William Li Yao as well as from the ordered dated
December 29, 1971 denying Li Yao's motion for reconsiderations.

William Li Yao, a Chinese national, filed a petition for naturalization on June 3, 1949 with the then Court of First
Instance of Manila, which petition was docketed as Case No. 8225. 1

After several hearings on the petition were held wherein the Office of the Solicitor General, in the
representation of the Republic of the Philippines appeared, the lower court rendered a decision dated October
25, 1950, the dispositive portion of which reads as follows:

IN VIEW OF ALL THE FOREGOING, the Court hereby declares William Li Yao, for all intents
and purposes a naturalized Filipino citizen, it appearing that he possesses all the qualifications
to become a naturalized Filipino and none of the disqualifications provided for by the law.
However, in view of the provisions of Republic Act No. 530, this decision shall not become final
and executory until after two (2) years from its promulgation and after this Court, on proper
hearing, with the attendance of the Solicitor General or his representative, is satisfied, and so
finds, that during the intervening time the applicant herein has (1) not left the Philippines, (2) has
dedicated himself continuously to a lawful calling or profession, (3) has not been convicted of
any offense and violation of the government promulgated rules, (4) or committed any act
prejudicial to the interest of the nation or contrary to any Government announce policies. After
the finding mentioned herein, this decision granting Philippine citizenship to the applicant herein
shall be registered and the oath provided by existing law shall be taken by said applicant,
whereupon, and not before, he will be entitled to all the privileges of the Filipino citizen and the
certificate of naturalization shall forthwith issue in his favor by the Clerk of this Court. 2

On November 20, 1952, acting on the petition of William Li Yao praying for the execution of the foregoing
decision and that he be allowed to take his oath of allegiance as a Filipino citizen, the lower court issued an
order, the dispositive portion of which reads as follows:

WHEREFORE, it appearing that the petitioner has complied, within the two year probation
period, with the provisions of Republic Act No. 530, he is hereby allowed to take his oath of
allegiance as a Filipino citizen, and Clerk of Court is directed to issue in his favor to the
corresponding certificate of naturalization. 3

About fifteen years later, or on January 5, 1968, the Republic of the Philippines, through the Solicitor General,
filed a motion to cancel William Li Yao's certificate of naturalization on the ground that it was fraudulently and
illegally obtained for the following reasons:

1. At the time of the filing of the petition, the applicant was not qualified to acquire Filipino
citizenship by naturalization because:

a. He was not a person of good moral character, having had illicit amorous
relationship (sic) with several women other than his lawfully wedded wife, by
whom he fathered illegitimate children (Li SiuLiat vs. Republic, L-25356,
November 25, 1967).

b. Nor had he conducted himself in an irreproachable manner in his dealings with


the duly constituted authorities:

(i) In contracting marriage, he used the name Fransisco Li Yao


(Exh. "J," p. 31, rec.)without prior judicial authority to use the
aforesaid first name Fransisco, the same not appearing to be his
baptismal name (Cosme Co Tian An vs. Republic, L-1983, August
31, 1966).

(ii) He was also known and had used the name and/or alias LI
CHAY TOO, JR. before the last World War, and under which
name, a trust fund was created for him (see Decision, Court of
Tax Appeals, CTA Case No. 30, dated July 31, 1956; also
Decision, Supreme Court, G.R. No. L-11861, Dec. 28, 1963).

(iii) He evaded the payment of lawful taxes due to the government


by underdeclaration of income as reflected in his income tax
returns for the years 1946-1951 (see Decision, Supreme Court,
William Li Yao v. Collector of Internal Revenue, L-11875,
December 28, 1963).

(iv) He committed violations of the Constitution and Anti-Dummy


laws prohibiting aliens from acquiring real properties by employing
dummies in the formation of a private domestic corporation, which
acquired the real properties.

(v) He made it appear, falsely, in the baptismal certificate of an


illegitimate son he fathered, named William Jose Antonio, that the
latter's mother is Juanita Tan Ho Ti, his law-mother is another
woman (sic). 4

William Li Yao opposed the forgoing motion on July 22, 1971. The lower court, however, without touching on
all the grounds upon which the said motion was based, relied solely on ground (iii) that William Li Yao evaded
the payment of lawful taxes due the government by underdeclaration of income as reflected in his income tax
returns for the years 1946-1951. It issued an order, the dispositive portion of which reads as follows:

WHEREFORE, the motion of the Republic of the Philippines to cancel Certificate of


Naturalization No. 1139 dated November 20, 1952 issued to the petitioner is hereby granted,
and the said Certificate of Naturalization should be, as it is hereby cancelled. Without
pronouncement as to cost. 5

6
William Li Yao filed a motion for reconsideration on December 29, 1971, which the lower court denied.

On January 7, 1972, William LI Yao filed a notice of appeal to this Court, manifesting that he was appealing
from the order of the lower court dated July 22, 1971, and from the order dated December 29, 1971. 7

After the parties had filed their respective briefs, petitioner-appellant Li Yao died. 8 The case has not, however,
become moot and academic since its disposition, either way, will have grave implications for the late petitioner-
appellant's wife and children.

The issue in this case is whether or not the cancellation of the certificate of naturalization of the deceased
petitioner-appellant William Li Yao made by the government through the Office of the Solicitor General is valid.

The appeal is without merit.

In his motion filed on January 5, 1968, the Solicitor General asked for the cancellation of the naturalization
certificate of appellant on the ground that it was "fraudelently and illegally obtained." This based on Section
18(a) of Com. Act No. 473, known as the Revised Naturalization Act, which provides that a naturalization
certificate may be cancelled "[i]f it is shown that said naturalization certificate was obtained fraudelently and
illegally."

It is indisputable that a certificate of naturalization may be cancelled if it is subsequently discovered that the
applicant therefore obtained it by misleading the court upon any material fact. 9 Law and jurisprudence even
authorize the cancellation of a certificate of naturalization upon grounds had conditions arising subsequent to
the granting of the certificate. 10 Moreover, a naturalization proceeding is not a judicial adversary proceeding,
the decision rendered therein, not constituting res judicata as to any matter that would support a judgment
cancelling a certificate of naturalization on the ground of illegal or fraudulent procurement thereof. 11

In ordering the cancellation of the naturalization certificate previously issued to appellant, the lower court
sustained the government's motion for cancellation on the sole finding that Li Yao had committed
underdeclaration of income and underpayment of income tax.

In the case entitled In the Matter of the Petition for Naturalization as Citizen of the Philippines, Lim Eng Yu
vs. Republic, 12 It was held that the concealment of applicant's income to evade payment of lawful taxes shows
that his moral character is not irreproachable, thus disqualifying him for naturalization.

Assuming arguendo, that appellant, as alleged, has fully paid or settled his tax liability under P.D. No. 68 which
granted a tax amnesty, such payment is not a sufficient ground for lifting the order of the lower court of July 22,
1971 cancelling his certificate of naturalization. The legal effect of payment under the decree is merely the
removal of any civil, criminal or administrative liability on the part of the taxpayer, only insofar as his tax case is
concerned. Thus, paragraph 4 of the decree provides;

4. That after full settlement of the accounts mentioned herein, the taxpayer shall be free of any
civil, criminal or administrative liability insofar as his tax case is involved (Emphasis supplied)

In other words, the tax amnesty does not have the effect of obliterating his lack of good moral character
and irreproachable conduct which are grounds for denaturalization.

The lower court based its order of cancellation of citizenship on the finding of evasion of payment of lawful
taxes which is sufficient ground, under Sec. 2 of the Revised Naturalization Law requiring, among others, that
applicant conduct himself "in a proper and irreproachable manner during the entire period of his residence in
the Philippines in his relation with constituted government as well as with the community in which he is
living," 13 to strip him of his citizenship without going into the other grounds for cancellation presented by the
Solicitor General.

Finally, taking into account the fact that naturalization laws should be rigidly enforced in favor of the
Government and against the applicant, this Court has repeatedly maintained the view that where the applicant
failed to meet the qualifications required for naturalization, the latter is not entitled to Filipino
citizenship. 14 More specifically, the Court has had occasion to state: "Admission to citizenship is one of the
highest privileges that the Republic of the Philippines can confer upon an alien. It is a privilege that should not
be conferred except upon persons fully qualified for it, and upon strict compliance with the law." 15 Philippine
citizenship is a pearl of great price which should be cherished and not taken for granted. Once acquired, its
sheen must be burnished and not stained by any wrongdoing which could constitute ample ground for
divesting one of said citizenship. Hence, compliance with all the requirements of the law must be proved to the
satisfaction of the Court. 16

WHEREFORE, the instant appeal is hereby DISMISSED and the assailed decision AFFIRMED.

SO ORDERED.

MOY YA LIM YAO VS. COMMISSIONER OF IMMIGRATION


G.R. No. L-21289, October 4 1971, 41 SCRA 292

FACTS:
Lau Yuen Yeung applied for a passport visa to enter the Philippines as a non-immigrant on 8 February 1961. In
the interrogation made in connection with her application for a temporary visitor's visa to enter the Philippines,
she stated that she was a Chinese residing at Kowloon, Hongkong, and that she desired to take a pleasure trip
to the Philippines to visit her great grand uncle, Lau Ching Ping. She was permitted to come into the
Philippines on 13 March 1961 for a period of one month.

On the date of her arrival, Asher Y. Cheng filed a bond in the amount of P1,000.00 to undertake, among
others, that said Lau Yuen Yeung would actually depart from the Philippines on or before the expiration of her
authorized period of stay in this country or within the period as in his discretion the Commissioner
ofImmigration or his authorized representative might properly allow.

After repeated extensions, Lau Yuen Yeung was allowed to stay in the Philippines up to 13 February 1962. On
25 January 1962, she contracted marriage with Moy Ya Lim Yao alias Edilberto Aguinaldo Lim an alleged
Filipino citizen. Because of the contemplated action of the Commissioner of Immigration to confiscate her bond
and order her arrest and immediate deportation, after the expiration of her authorized stay, she brought an
action for injunction. At thehearing which took place one and a half years after her arrival, it was admitted that
Lau Yuen Yeung could not write and speak either English or Tagalog, except for a few words. She could not
name any Filipino neighbor, with a Filipino name except one, Rosa. She did not know the names of her
brothers-in-law, or sisters-in-law. As a result, the Court of First Instance of Manila denied the prayer for
preliminary injunction. Moya Lim Yao and Lau Yuen Yeung appealed.

ISSUE:
Whether or not Lau Yuen Yeung ipso facto became a Filipino citizen upon her marriage to a Filipino citizen.

HELD:
Under Section 15 of Commonwealth Act 473, an alien woman marrying a Filipino, native born or naturalized,
becomes ipso facto a Filipina provided she is not disqualified to be a citizen of the Philippines under Section
4 of the same law. Likewise, an alien woman married to an alien who is subsequently naturalized here follows
the Philippine citizenship of her husband the moment he takes his oath as Filipino citizen, provided that she
does not suffer from any of the disqualifications under said Section 4. Whether the alien woman requires to
undergo the naturalization proceedings, Section 15 is a parallel provision to Section 16. Thus, if the widow of
an applicant for naturalization as Filipino, who dies during the proceedings, is not required to go through a
naturalization proceedings, in order to be considered as a Filipino citizen hereof, it should follow that the wife of
a living Filipino cannot be denied the same privilege.

This is plain common sense and there is absolutely no evidence that the Legislature intended to treat them
differently. As the laws of our country, both substantive and procedural, stand today, there is no
such procedure (a substitute for naturalization proceeding to enablethe alien wife of a Philippine citizen to have
the matter of her owncitizenship settled and established so that she may not have to be called upon to prove it
everytime she has to perform an act or enter into a transaction or business or exercise a right reserved only to
Filipinos), but such is no proof that the citizenship is not vested as of the date of marriage or the husband's
acquisition of citizenship, as the case may be, for the truth is that the situation obtains even as to native-born
Filipinos. Everytime the

Coquilla vs. COMELEC


Posted on October 3, 2012

G.R. No. 151914; 385 SCRA 607


September 17, 2002
____________________
Facts:
Petitioner Coquilla was born of Filipino parents in Oras, Eastern Samar, where he grew up and resided.
In 1965, he joined the US Navy and subsequently naturalized as a US citizen.

On October 15, 1998, petitioner came to the Philippines and took out a residence certificate, albeit continued
making several trips to the US.

On November 10, 2000, he took his oath as a citizen of the Philippines subsequently after his application for
repatriation was approved.

On November 21, 2000, he applied for registration as a voter of Butunga, Oras, Eastern Samar.
On February 27, 2001, he filed his COC stating therein that he has been a resident of Oras, Eastern Samar for
2 years.

On March 5, 2001, respondent incumbent mayor of Oras who was running for re-election, sought the
cancellation of petitioner’s COC on the ground that the latter had resided in Oras for only about 6 months since
when he took his oath as a citizen of the Philippines.

On May 14, 2001, petitioner garnered the highest number of votes and was subsequently proclaimed mayor of
Oras.

Issue:
WON petitioner satisfied the residency requirement for the position of mayor.
Held:
No. Par. 39, Chapter 1, Title 2 of the Local Government Code (RA 7160) provides that an elective official must
be a “…resident therein (barangay, municipality, city or province) for at least 1 year immediately preceeding
the day of the election…”
The term “residence” is to be understood not in its common acceptation as referring to “dwelling” or
“habitation,” but rather to “domicile” or legal residence, that is, “the place where a party actually or
constructively has his permanent home, where he, no matter where he may be found at any given time,
eventually intends to return and remain (animus manendi). A domicile of origin is acquired by every person at
birth. It is usually the place where the child’s parents reside and continues until the same is abandoned by
acquisition of a new domicile (domicile of choice).
In the case at bar, petitioner lost his domicile of origin in Oras by becoming a US citizen after enlisting in the
US Navy in 1965. From then on and until November 10, 2000, when he reacquired Philippine citizenship, he
was an alien.

Frivaldovs COMELEC [174 SCRA 245]


Posted by Pius Morados on November 6, 2011

(Municipal Corporation, Disqualification for Public Office)


Facts: Petitioner was proclaimed governor-elect of the province of Sorsogon on January 22, 1988. On October
27, 1988, respondents filed with the COMELEC a petition for the annulment of petitioner’s election and
proclamation on the ground that he was a naturalized American citizen and had not reacquired Philippine
citizenship on the day of the election on January 18, 1988. He was therefore not qualified to run for and be
elected governor.
Petitioner insisted that he was a citizen of the Philippines because his naturalization as an American citizen
was not “impressed with voluntariness.” His oath in his COC that he was a natural-born citizen should be a
sufficient act of repatriation. Additionally, his active participation in the 1987 congressional elections had
divested him of American citizenship under the laws of the US, thus restoring his Philippine citizenship.
The Solicitor General contends that petitioner was not a citizen of the Philippines and had not repatriated
himself after his naturalization as an American citizen. As an alien, he was disqualified for public office in the
Philippines. His election did not cure of this defect because the electorate could not amend the Constitution,
the Local Government Code and the Omnibus Election Code.

Issue: Whether or not petitioner was qualified to run for public office.
Held: No. First, petitioner’s loss of his naturalized American citizenship did not and could not have the effect of
automatic restoration of his Philippine citizenship.
Second, the mere filing of COC wherein petitioner claimed that he is a natural born Filipino citizen, is not a
sufficient act of repatriation.

Third, qualifications for public office are continuing requirements and must be possessed not only at the time
of appointment or election or assumption of office but during the officer’s entire tenure. Once any of the
required qualifications is lost, his title may be seasonably challenged

YU vs. DEFENSOR-SANTIAGO
GR No. L-83882, January 24, 1989

FACTS:

Petitioner Yu was originally issued a Portuguese passport in 1971. On February 10, 1978, he was naturalized
as a Philippine citizen. Despite his naturalization, he applied for and was issued Portuguese Passport by the
Consular Section of the Portuguese Embassy in Tokyo on July 21, 1981. Said Consular Office certifies that his
Portuguese passport expired on 20 July 1986. He also declared his nationality as Portuguese in commercial
documents he signed, specifically, the Companies registry of Tai Shun Estate Ltd. filed in Hongkong sometime
in April 1980.

The CID detained Yu pending his deportation case. Yu, in turn, filed a petition for habeas corpus. An internal
resolution of 7 November 1988 referred the case to the Court en banc. The Court en banc denied the petition.
When his Motion for Reconsideration was denied, petitioner filed a Motion for Clarification.

ISSUE:

Whether or not petitioner’s acts constitute renunciation of his Philippine citizenship

HELD:

Express renunciation was held to mean a renunciation that is made known distinctly and explicitly and not left
to inference or implication. Petitioner, with full knowledge, and legal capacity, after having renounced
Portuguese citizenship upon naturalization as a Philippine citizen resumed or reacquired his prior status as a
Portuguese citizen, applied for a renewal of his Portuguese passport and represented himself as such in
official documents even after he had become a naturalized Philippine citizen. Such resumption or reacquisition
of Portuguese citizenship is grossly inconsistent with his maintenance of Philippine citizenship.

While normally the question of whether or not a person has renounced his Philippine citizenship should be
heard before a trial court of law in adversary proceedings, this has become unnecessary as this Court, no less,
upon the insistence of petitioner, had to look into the facts and satisfy itself on whether or not petitioner's claim
to continued Philippine citizenship is meritorious.

Philippine citizenship, it must be stressed, is not a commodity or were to be displayed when required and
suppressed when convenient.

FRIVALDO VS. COMELEC (1996)


G.R. No. 120295, June 28 1996, 257 SCRA 727

FACTS:
Juan G. Frivaldo ran for Governor of Sorsogon again and won. Raul R. Lee questioned his citizenship. He then
petitioned for repatriation under Presidential Decree No. 725 and was able to take his oath of allegiance as a
Philippine citizen.

However, on the day that he got his citizenship, the Court had already ruled based on his previous attempts to
run as governor and acquire citizenship, and had proclaimed Lee, who got the secondhighest number of votes,
as the newly elect Governor of Sorsogon.

ISSUE:
Whether or not Frivaldo’s repatriation was valid.

HELD:
The Court ruled his repatriation was valid and legal and because of the curative nature of Presidential Decree
No. 725, his repatriation retroacted to the date of the filing of his application to run for governor. The steps to
reacquire Philippine Citizenship by repatriation under Presidential Decree No. 725 are: (1) filing theapplication;
(2) action by the committee; and (3) taking of the oath of allegiance if the application is approved. It is only
upon taking the oath of allegiance that the applicant is deemed ipso jure to have reacquired Philippine
citizenship. If the decree had intended the oath taking to retroact to the date of the filing of the application, then
it should not have explicitly provided otherwise. He is therefore qualified to be proclaimed governor of
Sorsogon.

ALTAREJOS VS COMELEC
Posted by kaye lee on 9:25 PM

G.R. No. 163256, 10 Nov 2004 [Naturalization; Reacquisition]

FACTS:
Private respondents filed with the COMELEC to disqualify and deny due course or cancel the certificate of
candidacy of Ciceron P. Altarejos, on the ground that he is not a Filipino citizen and that he made a false
representation in his COC that he was not a permanent resident of the Municipality of San Jacinto, Masbate,
the town he's running for as mayor in the May 10, 2004 elections. Altarejos answered that he was already
issued a Certificate of Repatriation by the Special Committee on Naturalization in December 17, 1997.
ISSUE:
Whether or not the registration of petitioner’s repatriation with the proper civil registry and with the Bureau of
Immigration a prerequisite in effecting repatriation.

RULING:
Yes. The registration of certificate of repatriation with the proper local civil registry and with the Bureau of
Immigration is a prerequisite in effecting repatriation. Petitioner completed all the requirements of repatriation
only after he filed his certificate of candidacy for a mayoralty position but before the elections. Petitioner’s
repatriation retroacted to the date he filed his application and was, therefore, qualified to run for a mayoralty
position in the government in the May 10, 2004 elections.

ANGAT vs. REPUBLIC


G.R. No. 132244, September 14, 1999

FACTS:

Petitioner Gerardo Angat was a natural born citizen of the Philippines until he lost hiscitizenship by
naturalization in the United States of America. On 11 March 1996, he filedbefore the RTC of Marikina City,
Branch 272, a petition to regain his Status as a citizen of the Philippines under Commonwealth Act No. 63,
Republic Act No. 965 and Republic Act No. 2630. The case was thereafter set for initial hearing.

On 13 June 1996, petitioner sought to be allowed to take his oath of allegiance to the Republic of the
Philippines pursuant to R.A. 8171. The motion was initially denied by the trial judge but after a motion for
reconsideration, it was granted. The petitioner was ordered to take his oath of allegiance pursuant to R.A.
8171. After taking his oath of allegiance, the trial court issued an order repatriating petitioner and declaring him
as citizen of the Philippines pursuant to Republic Act No. 8171. The Bureau of Immigration was ordered
to cancel his alien certificate of registration and issue the certificate of identification as Filipino citizen.

On 19 March 1997, the Office of the Solicitor General filed a Manifestation and Motion (virtually a motion for
reconsideration) asserting that the petition itself should have been dismissed by the court a quo for lack of
jurisdiction because the proper forum for it was the Special Committee on Naturalization consistently with
Administrative Order No. 285 ("AO 285"), dated 22 August 1996, issued by President Fidel V. Ramos. AO 285
had tasked the Special Committee on Naturalization to be the implementing agency of R.A 8171. The trial
court granted the motion and dismissed the petition.

Petitioner appealed contending that the RTC seriously erred in dismissing thepetition by giving retroactive
effect to Administrative Order No. 285, absent a provision on Retroactive Application.

ISSUES:

WON Court erred in dismissing the petition by giving retroactive effect to AO 285, absent a provision on
Retroactive Application
HELD:

No. Under Section 1 of Presidential Decree ("P.D.") No. 725, 8 dated 05 June 1975, amending Commonwealth
Act No. 63, an application for repatriation could be filed by Filipino women who lost their
Philippine citizenship by marriage to aliens, as well as by natural born Filipinos who lost their
Philippine citizenship, with the Special Committee on Naturalization. The committee, chaired by the Solicitor
General with the Undersecretary of Foreign Affairs and the Director of the National Intelligence Coordinating
Agency as the other members, was created pursuant to Letter of Instruction ("LOI") No. 270, dated 11 April
1975, as amended by LOI No. 283 and LOI No. 491 issued, respectively, on 04 June 1975 and on 29
December 1976. Although the agency was deactivated by virtue of President Corazon C. Aquino's
Memorandum of 27 March 1987, it was not however, abrogated. In Frivaldo vs. Commission on Elections, 9
the Court observed that the aforedated memorandum of President Aquino had merely directed the Special
Committee on Naturalization "to cease and desist from undertaking any and all proceedings . . . under Letter of
Instruction ("LOI") 270." 10 The Court elaborated:

This memorandum dated March 27, 1987 cannot by any stretch of legal hermeneutics be construed as a law
sanctioning or authorizing a repeal of P.D. No. 725. Laws are repealed only by subsequent ones and a repeal
may be express or implied. It is obvious that no express repeal was made because then President Aquino in
her memorandum-based on the copy furnished us by Lee-did not categorically and/or impliedly state that P.D.
725 was being repealed or was being rendered without any legal effect. In fact, she did not even mention it
specifically by its number or text. On the other hand, it is a basic rule of statutory construction that repeals by
implication are not favored. An implied repeal will not be allowed "unless it is convincingly and unambiguously
demonstrated that the two laws are clear repugnant and patently inconsistent that they cannot co-exist."

Indeed, the Committee was reactivated on 08 June 1995; hence, when petitioner filed his petition on 11 March
1996, the Special Committee on Naturalizationconstituted pursuant to LOI No. 270 under P.D. No. 725 was in
place. Administrative Order 285, promulgated on 22 August 1996 relative to R.A. No. 8171, in effect, was
merely then a confirmatory issuance.

The Office of the Solicitor General was right in maintaining that Angat's petitionshould have been filed with the
Committee, aforesaid, and not with the RTC which had no jurisdiction thereover. The court's order of 04
October 1996 was thereby null and void, and it did not acquire finality nor could be a source of right on the part
of petitioner.

It should also be noteworthy that the was one for repatriation, and it was thusincorrect for petitioner to initially
invoke Republic Act No. 965 and R.A. No. 2630 since these laws could only apply to persons who had lost
their citizenship by rendering service to, or accepting commission in, the armed forces of an alliedforeign
country or the armed forces of the United States of America, a factual matter not alleged in the petition,
Parenthetically, under these statutes, the person desiring to re-acquire Philippine citizenship would not even be
required to file a petition in court, and all that he had to do was to take an oath of allegianceto the Republic of
the Philippines and to register that fact with the civil registry in the place of his residence or where he had last
resided in the Philippines.
G.R. No. L-29397 March 29, 1983

MODESTA DUGCOY JAO (SOMETIMES SPELLED HAO), petitioner,


vs.
REPUBLIC OF THE PHILIPPINES, oppositor.

Julius A Artes for petitioner-appellee.

The Solicitor General for oppositor-appellant.

RESOLUTION

VASQUEZ, J.:

The petitioner filed a petition in the Court of First Instance of Davao for repatriation under Commonwealth Act
No. 63, as amended, alleging therein that although her father was Chinese, she was a citizen of the Philippines
because her mother was a Filipina who was not legally married to her Chinese husband; that she lost her
Philippine citizenship when she married Go Wan, a Chinese, with whom she had three children; and that Go
Wan died in September 6, 1962. Petitioner further alleged that her illiterate mother erroneously registered her
as an alien with the Bureau of Immigration, by virtue of which she was issued Alien Certificate of Registration
No. A-176678. The petition contains no prayer for relief.

The petition was not published, but notice thereof was served on the Provincial Fiscal who appeared at the
hearing but presented no evidence. After receiving the evidence of the petitioner, the trial court issued an
Order declaring the petitioner as "judicially repatriated," and ordering the cancellation of her alien certificate of
registration. The Provincial Fiscal, in behalf of the Republic of the Philippines, took this appeal.

The proceedings taken in the trial court are a complete nullity. There is no law requiring or authorizing that
repatriation should be effected by a judicial proceeding. All that is required for a female citizen of the
Philippines who lost her citizenship to an alien to reacquire her Philippine citizen, upon the termination of her
marital status, "is for her to take necessary oath of allegiance to the Republic of the Philippines and to register
the said oath in the proper civil registry" (Lim vs. Republic, 37 SCRA 783). Moreover, the petitioner's claim of
Philippine citizenship prior to her marriage for being allegedly an illegitimate child of a Chinese father and a
Filipino mother may not be established in an action where the mother or her heirs are not patties (Tan Pong vs.
Republic, 30 Phil. 380). It is the consistent rule in this jurisdiction that Philippine citizenship may not be
declared in a non- adversary suit where the persons whose rights are affected by such a declaration are not
parties, such as an action for declaratory relief (Tiu Navarro vs. Commissioner of Immigration, 107 Phil. 632) a
petition for judicial repatriation (Lim vs. Republic, supra), or an action to cancel registration as an alien (Tan vs.
Republic, L-16108, Oct. 31, 1961). As was held in Lim vs. Republic, supra, "there is no proceeding established
by law or the rules by which any person claiming to be a citizen may get a declaration in a court of justice to
that effect or in regard to his citizenship."

ACCORDINGLY, the decision appealed from is hereby revoked and set aside.

SO ORDERED.

NICOLAS-LEWIS vs. COMELEC


G.R. No. 162759, August 4, 2006
Facts:

Petitioners are successful applicants for recognition of Philippine citizenship under R.A. 9225, the Citizenship
Retention and Re-Acquisition Act of 2003. Long before the May 2004 national and local elections, petitioners
sought registration and certification as "overseas absentee voter" only to be advised by the Philippine
Embassy in the United States that, per a COMELEC letter to the Department of Foreign Affairs dated
September 23, 2003, they have yet no right to vote in such elections owing to their lack of the one-year
residence requirement prescribed by the Constitution.

Faced with the prospect of not being able to vote in the May 2004 elections owing to the COMELEC's refusal
to include them in the National Registry of Absentee Voters, petitioner Nicolas-Lewis et al., filed on April 1,
2004 a petition for certiorari and mandamus, praying that they and others who retained or reacquired Philippine
citizenship under No. 9225, be allowed to avail themselves of the mechanism provided under the Overseas
Absentee Voting Act of 2003 (R.A. 9189) and that the COMELEC accordingly be ordered to allow them to vote
and register as absentee voters under the aegis of R.A. 9189.

On April 30, 2004 (a little over a week before Election Day), COMELEC filed a Comment praying for the denial
of the petition. Consequently, petitioners were not able to register let alone vote in said elections.

On May 20, 2004, the OSG filed a Manifestation (in Lieu of Comment) stating that “all qualified overseas
Filipinos, including dual citizens who care to exercise the right of suffrage, may do so,” observing, however,
that the conclusion of the 2004 elections had rendered the petition moot and academic.

Issues:

1. Must the Supreme Court still resolve said petition considering that under the circumstances the same has
already been rendered moot and academic?

2. WON dual citizens may exercise their right to suffrage as absentee voters even short of the 1-yr residency
requirement.

Held:

1. Yes. The holding of the 2004 elections had indeed rendered the petition moot and academic, but only
insofar as petitioners’ participation in such political exercise is concerned. The broader and transcendental
issue tendered in the petition is the propriety of allowing dual citizens to participate and vote as absentee voter
in future elections, which however, remains unresolved.

2. Yes. We start off with Sections 1 and 2 of Article V of the Constitution, respectively reading as follows:

SEC 1. Suffrage may be exercised by all citizens of the Philippines not otherwise disqualified by law, who are
at least eighteen years of age, and who shall have resided in the Philippines for at least one year and in the
place wherein they propose to vote for at least six months immediately preceding the election. xxx.
SEC 2. The Congress shall provide … a system for absentee voting by qualified Filipinos abroad.

In a nutshell, the aforequoted Section 1 prescribes residency requirement as a general eligibility factor for the
right to vote. On the other hand, Section 2 authorizes Congress to devise a system wherein an absentee may
vote, implying that a non-resident may, as an exception to the residency prescription in the preceding section,
be allowed to vote.

In response to its above mandate, Congress enacted R.A. 9189 - the OAVL - identifying in its Section 4 who
can vote under it and in the following section who cannot.

Soon after Section 5(d) of R.A. 9189 passed the test of constitutionality, Congress enacted R.A. 9225 the
relevant portion of which reads:

SEC. 5.Civil and Political Rights and Liabilities. – Those who retain or re-acquire Philippine citizenship
under this Act shall enjoy full civil and political rights and be subject to all attendant liabilities and
responsibilities under existing laws of the Philippines and the following conditions:

(1) Those intending to exercise their right of suffrage must meet the requirements under Section 1, Article V of
the Constitution, Republic Act No. 9189, otherwise known as “The Overseas Absentee Voting Act of 2003” and
other existing laws; xxx

There is no provision in the dual citizenship law - R.A. 9225 - requiring "duals" to actually establish residence
and physically stay in the Philippines first before they can exercise their right to vote. On the contrary, R.A.
9225, in implicit acknowledgment that “duals” are most likely non-residents, grants under its Section 5(1) the
same right of suffrage as that granted an absentee voter under R.A. 9189. It cannot be overemphasized that
R.A. 9189 aims, in essence, to enfranchise as much as possible all overseas Filipinos who, save for the
residency requirements exacted of an ordinary voter under ordinary conditions, are qualified to vote.

Considering the unison intent of the Constitution and R.A. 9189 and the expansion of the scope of that law with
the passage of R.A. 9225, the irresistible conclusion is that "duals" may now exercise the right of suffrage thru
the absentee voting scheme and as overseas absentee voters. R.A. 9189 defines the terms adverted to in the
following wise:

“Absentee Voting” refers to the process by which qualified citizens of the Philippines abroad exercise their right
to vote;

“Overseas Absentee Voter” refers to a citizen of the Philippines who is qualified to register and vote under this
Act, not otherwise disqualified by law, who is abroad on the day of elections;

While perhaps not determinative of the issue tendered herein, we note that the expanded thrust of R.A.
9189 extends also to what might be tag as the next generation of "duals". This may be deduced from the
inclusion of the provision on derivative citizenship in R.A. 9225 which reads:
SEC. 4.Derivative Citizenship. – The unmarried child, whether legitimate, illegitimate or adopted, below
eighteen (18) years of age, of those who re-acquire Philippine citizenship upon effectivity of this Act shall be
deemed citizens of the Philippines.

It is very likely that a considerable number of those unmarried children below eighteen (18) years
of age had never set foot in the Philippines. Now then, if the next generation of "duals" may nonetheless
avail themselves the right to enjoy full civil and political rights under Section 5 of the Act, then there is neither
no rhyme nor reason why the petitioners and other present day "duals," provided they meet the requirements
under Section 1, Article V of the Constitution in relation to R.A. 9189, be denied the right of suffrage as an
overseas absentee voter. Congress could not have plausibly intended such absurd situation.

Aznar vs .COMELEC
Post under case digests, Political Law at Saturday, February 04, 2012 Posted by Schizophrenic Mind

Facts: Aznar filed a petition for certiorari to review COMELEC resolution proclaiming Osmena as the Cebu
Governor. He alleged that Osmena is an American thus disqualified to run in the 1988 election. He presented
evidence. Osmena claimed that he is a Filipino.

Issue: Whether or not Osmena is an American thus disqualified torun in the elections

Held: No . No substantial & convincing evidence presented to prove Osmena is no longer a Filipino citizen &
disqualified from running. Filipino citizenship is lost by naturalization in a foreign country or by express
renunciation of citizenship or by subscribing to an oath of allegiance to support another country’s constitution
or laws (CA No. 63). No proof Osmena did any of those. Aznar assumed that the ACR & permit to re-enter
were proof of such. However, only RP courts are allowed to determine whether one is a Filipino citizen or not,
regardless of whether that person is considered an American under US laws. His father is Filipino thus, without
proof to the contrary, the presumption that he is a Filipino remains.
MERCADO VS. MANZANO
G.R. No. 135083, May 26 1999

FACTS:
Petitioner Ernesto S. Mercado and private respondent Eduardo B. Manzano were candidates for vice mayor of
the City of Makati in the May 11, 1998 elections. Respondent was then declared the winning candidate;
however its proclamation was suspended in view of a pending petition for disqualification filed by a certain
Ernesto Mamaril who alleged that private respondent was not a citizen of the Philippines but of the United
States.

In its resolution, dated May 7, 1998, the Second Division of the COMELEC granted the petition of Mamaril and
ordered the cancellation of the certificate of candidacy of private respondent on the ground that he is a dual
citizen and, Section 40(d) of the Local Government Code provides that persons with dual citizenship are
disqualified from running for any elective position.

Respondent admitted that he is registered as a foreigner with the Bureau of Immigration under Alien Certificate
of Registration No. B-31632 and alleged that he is a Filipino citizen because he was born in 1955 of a Filipino
father and a Filipino mother. He was born in the United States, San Francisco, California, on September 14,
1955, and is considered an American citizen under US Laws. But notwithstanding his registration as an
American citizen, he did not lose his Filipino citizenship. From these facts, respondent is a dual citizen - both a
Filipino and a US citizen.

ISSUE:
Whether or not Manzano is qualified to hold office as Vice-Mayor.

HELD:
The petition was dismissed. Dual citizenship is different from dual allegiance. The former arises when, as a
result of the concurrent application of the different laws of two or more states, a person is simultaneously
considered a national by the said states. For instance, such a situation may arise when a person whose
parents are citizens of a state which adheres to the principle of jus sanguinis is born in a state which follows
the doctrine of jus soli. Private respondent is considered as a dual citizen because he is born of Filipino
parents but was born in San Francisco, USA. Such a person, ipso facto and without any voluntary act on his
part, is concurrently considered a citizen of both states. Considering the citizenship clause under Article IV of
the Constitution, it is possible for the following classes of citizens of the Philippines to posses dual citizenship:
(1) Those born of Filipino fathers and/or mothers in foreign countries which follow the principle of jus soli;

(2) Those born in the Philippines of Filipino mothers and alien fathers if by the laws of their fathers’ country
such children are citizens of that country;
(3) Those who marry aliens if by the laws of the latter’s country the former are considered citizens, unless by
their act or omission they are deemed to have renounced Philippine citizenship.
Dual allegiance, on the other hand, refers to the situation in which a person simultaneously owes, by some
positive act, loyalty to two or more states. While dual citizenship is involuntary, dual allegiance is the result of
an individual’s volition.

By filing a certificate of candidacy when he ran for his present post, private respondent elected Philippine
citizenship and in effect renounced his American citizenship. The filing of such certificate of candidacy sufficed
to renounce his American citizenship, effectively removing any disqualification he might have as a dual citizen.

By declaring in his certificate of candidacy that he is a Filipino citizen; that he is not a permanent resident or
immigrant of another country; that he will defend and support the Constitution of the Philippines and bear true
faith and allegiance thereto and that he does so without mental reservation, private respondent has, as far as
the laws of this country are concerned, effectively repudiated his American citizenship and anything which he
may have said before as a dual citizen. On the other hand, private respondent’s oath of allegiance to the
Philippine, when considered with the fact that he has spent his youth and adulthood, received his education,
practiced his profession as an artist, and taken part in past elections in this country, leaves no doubt of his
election of Philippine citizenship.

Valles vs. COMELEC


G.R. No. 137000, Aug. 9, 2000

 Principle of jus sanguinis


 How Philippine citizenship is acquired
 Effect of filing certificate of candidacy: express renunciation of other citizenship

FACTS:

Rosalind Ybasco Lopez was born on May 16, 1934 in Australia to a Filipino father and an Australian mother. In
1949, at the age of fifteen, she left Australia and came to settle in the Philippines, where she later married a
Filipino and has since then participated in the electoral process not only as a voter but as a candidate, as well.
In the May 1998 elections, she ran for governor but Valles filed a petition for her disqualification as candidate
on the ground that she is an Australian.

ISSUE:

 Whether or not Rosalind is an Australian or a Filipino

HELD:

The Philippine law on citizenship adheres to the principle of jus sanguinis. Thereunder, a child follows the
nationality or citizenship of the parents regardless of the place of his/her birth, as opposed to the doctrine of jus
soli which determines nationality or citizenship on the basis of place of birth.

Rosalind Ybasco Lopez was born a year before the 1935 Constitution took into effect and at that time, what
served as the Constitution of the Philippines were the principal organic acts by which the United States
governed the country. These were the Philippine Bill of July 1, 1902 and the Philippine Autonomy Act of Aug.
29, 1916, also known as the Jones Law.

Under both organic acts, all inhabitants of the Philippines who were Spanish subjects on April 11, 1899 and
resided therein including their children are deemed to be Philippine citizens. Private respondents father,
TelesforoYbasco, was born on Jan. 5, 1879 in Daet, Camarines Norte.... Thus, under the Philippine Bill of 1902
and the Jones Law, TelesforoYbasco was deemed to be a Philippine citizen. By virtue of the same laws, which
were the laws in force at the time of her birth, Telesforo’s daughter, herein private respondent Rosalind
YbascoLopez, is likewise a citizen of the Philippines.

The signing into law of the 1935 Philippine Constitution has established the principle of jus sanguinis as basis
for the acquisition of Philippine citizenship, xxx

So also, the principle of jus sanguinis, which confers citizenship by virtue of blood relationship, was
subsequently retained under the 1973 and 1987 Constitutions. Thus, the herein private respondent, Rosalind
Ybasco Lopez, is a Filipino citizen, having been born to a Filipino father. The fact of her being born in Australia
is not tantamount to her losing her Philippine citizenship. If Australia follows the principle of jus soli, then at
most, private respondent can also claim Australian citizenship resulting to her possession of dual citizenship.

Das könnte Ihnen auch gefallen